Difference between revisions of "2002 AMC 12A Problems/Problem 2"

 
(7 intermediate revisions by 3 users not shown)
Line 1: Line 1:
 +
{{duplicate|[[2002 AMC 12A Problems|2002 AMC 12A #2]] and [[2002 AMC 10A Problems|2002 AMC 10A #6]]}}
 +
 
==Problem==
 
==Problem==
  
 
Cindy was asked by her teacher to subtract 3 from a certain number and then divide the result by 9. Instead, she subtracted 9 and then divided the result by 3, giving an answer of 43. What would her answer have been had she worked the problem correctly?
 
Cindy was asked by her teacher to subtract 3 from a certain number and then divide the result by 9. Instead, she subtracted 9 and then divided the result by 3, giving an answer of 43. What would her answer have been had she worked the problem correctly?
  
<math> \mathrm{(A) \ } 15\qquad \mathrm{(B) \ } 34\qquad \mathrm{(C) \ } 43\qquad \mathrm{(D) \ } 51\qquad \mathrm{(E) \ } 138 </math>
+
<math> \textbf{(A) } 15\qquad \textbf{(B) } 34\qquad \textbf{(C) } 43\qquad \textbf{(D) } 51\qquad \textbf{(E) } 138 </math>
 +
 
 +
==Solution==
 +
 
 +
We work backwards; the number that Cindy started with is <math>3(43)+9=138</math>. Now, the correct result is <math>\frac{138-3}{9}=\frac{135}{9}=15</math>. Our answer is <math>\boxed{\textbf{(A) }15}</math>.
  
  
==Solution==
+
==Solution 2==
 +
 
 +
Let the number be <math>x</math>. We transform the problem into an equation: <math>\frac{x-9}{3}=43</math>. Solve for <math>x</math> gives us <math>x=138</math>. Therefore, the correct result is <math>\frac{138-3}{9}=\frac{135}{9}=\boxed{\textbf{(A) }15}</math>.
  
<math>\dfrac{x-9}{3}=43</math>
+
==Video Solution by Daily Dose of Math==
  
<math>x=43*3+9=138</math>
+
https://youtu.be/ZgEkIB1qmRw?si=LpBb-qb3OaUbQ8_W
  
<math>\dfrac{x-3}{9}=15 \Rightarrow \mathrm {(A)}</math>
+
~Thesmartgreekmathdude
  
 
==See Also==
 
==See Also==
  
 
{{AMC12 box|year=2002|ab=A|num-b=1|num-a=3}}
 
{{AMC12 box|year=2002|ab=A|num-b=1|num-a=3}}
 +
{{AMC10 box|year=2002|ab=A|num-b=5|num-a=7}}
 +
{{MAA Notice}}

Latest revision as of 11:41, 21 July 2024

The following problem is from both the 2002 AMC 12A #2 and 2002 AMC 10A #6, so both problems redirect to this page.

Problem

Cindy was asked by her teacher to subtract 3 from a certain number and then divide the result by 9. Instead, she subtracted 9 and then divided the result by 3, giving an answer of 43. What would her answer have been had she worked the problem correctly?

$\textbf{(A) } 15\qquad \textbf{(B) } 34\qquad \textbf{(C) } 43\qquad \textbf{(D) } 51\qquad \textbf{(E) } 138$

Solution

We work backwards; the number that Cindy started with is $3(43)+9=138$. Now, the correct result is $\frac{138-3}{9}=\frac{135}{9}=15$. Our answer is $\boxed{\textbf{(A) }15}$.


Solution 2

Let the number be $x$. We transform the problem into an equation: $\frac{x-9}{3}=43$. Solve for $x$ gives us $x=138$. Therefore, the correct result is $\frac{138-3}{9}=\frac{135}{9}=\boxed{\textbf{(A) }15}$.

Video Solution by Daily Dose of Math

https://youtu.be/ZgEkIB1qmRw?si=LpBb-qb3OaUbQ8_W

~Thesmartgreekmathdude

See Also

2002 AMC 12A (ProblemsAnswer KeyResources)
Preceded by
Problem 1
Followed by
Problem 3
1 2 3 4 5 6 7 8 9 10 11 12 13 14 15 16 17 18 19 20 21 22 23 24 25
All AMC 12 Problems and Solutions
2002 AMC 10A (ProblemsAnswer KeyResources)
Preceded by
Problem 5
Followed by
Problem 7
1 2 3 4 5 6 7 8 9 10 11 12 13 14 15 16 17 18 19 20 21 22 23 24 25
All AMC 10 Problems and Solutions

The problems on this page are copyrighted by the Mathematical Association of America's American Mathematics Competitions. AMC logo.png